Vous êtes sur la page 1sur 36

Workshop

on
Electrolyte and
Acid-Base Disturbances

Organisers
AK Agarwal, RK Singal

Editor
Praveen Aggarwal
Governing Body of Association of Physicians of India

President Elect President Past President


AK Agarwal, New Delhi (2009) SK Bichile, Mumbai (2009) RK Singal, New Delhi (2009)

Vice Presidents
BR Bansode, Mumbai (2009) Pritam Gupta, New Delhi (2010) Alaka Deshpande, Mumbai (2011)

Hon. General Secretary


Sandhya Kamath, Mumbai (2010)

Hon.Treasurer
Milind Y Nadkar, Mumbai (2009)

Members
Shyam Sundar, Varanasi (2009) Rohini Handa, New Delhi (2010) Narinder Pal Singh, New Delhi (2011)
Y Satyanarayan Raju, Hyderabad (2009) Amal Kumar Banerjee, Howrah (2010) Madhukar Rai, Varanasi (2011)
J Mukhopadhyay, Howrah (2009) R Sajithkumar, Kottayam (2010) Surendra Daga, Kolkata (2011)
NK Singh, Dhanbad (2009) Abhay N Rai, Gaya (2010) S Arulrhaj, Tuticorin (2011)

Zonal Members
North Zone Rajesh Upadhyay, New Delhi (2011) Mid South Zone G Narsimulu, Hyderabad (2011)
NorthWest Zone Gurpreet SWander, Ludhiana (2011) South Zone A Muruganathan, Tirupur (2011)
Central Zone Sanjiv Maheshwari, Ajmer (2011) Mid East Zone KamleshTewary, Muzaffarpur (2011)
West Zone Ashit Bhagwati, Mumbai (2011) East Zone Samar Kumar Banerjee, Kolkata (2011)

Ex Officio Member
Dean ICP
AK Das, Pondicherry

Invited Members
Hon.Editor,JAPI API House Chairman Editor-in-chief,APIText Book
Shashank R Joshi, Mumbai Siddharth N Shah, Mumbai YP Munjal, New Delhi

Co-opted Members
Jt.Secretary, Presidents Place Armed Forces, Medical Services Jt.Secretary (HQ)
JM Phadtare, Mumbai (2009) Lt.Gen.SR Mehta, New Delhi (2007-2008) Falguni Parikh, Mumbai

Organising Secretary, APICON 2008 Organising Secretary, APICON 2008


NN Asokan, Muvattupuzha NK Soni, Ghaziabad
APICON 2009
Workshop on Electrolyte and Acid-Base Disturbances

Date: January 30, 2009

Time: 2.00 pm 6.00 pm

Convener
Praveen Aggarwal, All India Institute of Medical Sciences
New Delhi

Faculty
l Praveen Aggarwal, New Delhi
l Chhavi Sawhney, New Delhi
l Sanjeev Bhoi, New Delhi

Electrolyte and Acid-Base Disturbances l APICON 2009 l Jan. 29 - Feb. 1, 2009 1


SCIENTIFIC COMMITTEE, APICON 2009

A. K. Agarwal
Chairman, Scientific Committee
S. K. Bichile
President, Association of Physicians of India (API)
A. K. Das
Dean, Indian College of Physicians (ICP)
B. B. Thakur
Dean-Elect, ICP
Sandhya Kamath
Hony. General Secretary, API and ICP
Shashank R. Joshi
Hony. Editor, Journal of Association of Physicians of India (JAPI)
N. K. Soni
Organising Secretary, APICON - 2009

Members Scientific Committee


R. K. Singal Surendra K. Sharma Samar Banerjee Shyam Sundar

Principal Advisors
Y. P. Munjal B. K. Sahay Siddharth N. Shah

Members Advisory Board


S. Arulrhaj T. Kadhiravan J. M. Phadtare
Amal Kumar Banerjee O. P. Kalra Prashant Prakash
B. R. Bansode Ulka Kamble A. N. Rai
Subhash Chandra Umesh Kansra Y. Satyanarayan Raju
Anil Chaturvedi Ram Kapoor B. B. Rewari
S. Chugh V. N. Kaushal Anita Sharma
Naveen Garg Navin Kumar N. K. Singh
P. S. Ghalaut Rajat Kumar N. P. Singh
Anil Gomber Sanjiv Maheshwari Rita Sood
Vinay Gulati Lt. Gen. S. R. Mehta Dinesh Srivastava
B. Gupta Vipin Mendiratta B. K. Tripathi
B. B. Gupta Adhip Mitra Rajesh Upadhyay
Pritam Gupta Alladi Mohan A. K. Vaish
Rohini Handa Ashutosh Mohan A. K. Varshney

2 Electrolyte and Acid-Base Disturbances l APICON 2009 l Jan. 29 - Feb. 1, 2009


Contents

1. Fluid and Electrolyte Disturbances 4


Sanjeev Bhoi, Chhavi Sawhney

2. Acid-Base Disturbances 19
Praveen Aggarwal

Electrolyte and Acid-Base Disturbances l APICON 2009 l Jan. 29 - Feb. 1, 2009 3


1. Fluid and Electrolyte Disturbances
Sanjeev Bhoi, Chhavi Sawhney

The Body Water sweat, exudates, and transudates can also be considered
as specialized portions of the extracellular water, because
About a billion years ago, life began-in the sea. The sea when these are lost a severe loss of extracellular water
possessed unique properties for the maintenance of life. occurs.
For example, the water of the sea is a solvent for the
electrolytes and the oxygen that are necessary for life. The relation of the body water to the body weight are as
The sea is also a solvent for the carbon dioxide that follows:
accumulates during lifes processes. Since the carbon
Water Compartment Percentage Volume in
dioxide is volatile, it can be easily dissipated from the
of Body Liters
surface of the sea. In addition, the volume of the sea is
Weight (man, 70 kg)
so great that it can absorb large amounts of heat, or lose
large amounts of heat, with only relatively small changes Plasma water (plus) 4 2.8
in temperature. The volume of the sea is also so great Interstitial water 16 11.2
that significant changes in its composition occur only
over a period of hundreds of thousands of years. Finally, Total extracellular
its dielectric constant, its surface tension, and other water (plus) = 20 = 14
physical properties are all important in maintaining and Intracellular water 40 28
protecting life. Total average body
As a result, the water that surrounds the cells of water in a man = 60 = 42
vertebrates and of humans, namely, the extracellular Total average body
water, still has an electrolyte composition similar to what water in a 70 kg woman 50 35
the sea had in pre-recorded times, in spite of all the
countless changes in evolution that have occurred. Over These figures represent average values that we shall use
the years, the rivers of the world have eroded land and in calculating water and electrolyte requirements in this
washed elements into the sea. This has caused the book.
electrolytes of the sea to become more concentrated. In The total body water varies. It is related principally to
spite of this, there is still a remarkable similarity between the fat content of the body, and to sex. Fat has relatively
the proportional composition of electrolytes in seawater less water associated with it; therefore a fat person will
and in extracellular water. have relatively less water than a thin person. In addition,
The total amount of water in the body can be determined a woman has a lower content of body water than a man.
by introducing into the body a known quantity of a The water content of the body also decreases with age.
substance that diffuses evenly throughout the The average water content of a man is approximately
extracellular water and the cells, and then determining 60%. The average water content of a woman is
its concentration. Antipyrine, urea, thiourea, and more approximately 50%.
recently heavy water (deuterium oxide or tritium
oxide) have been used for this purpose. Body water is also composed of inaccessible bone water
and transcellular fluids.
The Extracellular Water Transcellular fluids include the fluid of organs such as
the kidneys, liver, pancreas, skin and the mucous
The body water is usually divided into two main
membranes of the gastrointestinal and respiratory, tracts;
compartments: the extracellular water (extracellular
and so on. It is difficult to measure the volume of the
fluid) and the intracellular water (intracellular fluid).
fluid in these compartments.
The extracellular water includes the plasma water and
the interstitial water (the fluid in the tissue spaces, lying Body water, including percentage of fat and of lean body
between the cells). Gastrointestinal secretions, urine, tissue, can now be measured using a bioelectrical

4 Electrolyte and Acid-Base Disturbances l APICON 2009 l Jan. 29 - Feb. 1, 2009


impedance analyzer. This is due to the fact that the capillary walls prevent
the outward movement of most of the protein from
The Electrolytes in the Extracellular Water the plasma. As a result, the sodium concentration in
plasma is slightly greater than the sodium
Chemists are able to determine the nature and the concentration in the interstitial water. Conversely, the
concentration of the electrolytes in both the extracellular chloride concentration in the plasma is slightly less
water and the cells. However, the determination of than the chloride concentration in the interstitial water.
electrolyte concentration in the cells requires special
research techniques. Therefore, physicians must rely on The electrolyte concentrations are given in terms of
the changes in electrolyte concentration in the milliequivalents per liter of serum or plasma (mEq/L).
extracellular water, particularly in the plasma or serum,
in treating patients. Milliequivalents
The relative concentration of electrolytes in the The electrolytes of the body are in solution, mostly in
extracellular water and in the cells is shown in Table 1. the form of ions. Their concentration can be described
Notice that sodium (Na) and chloride (Cl) are the in terms of weight, such as milligrams (mg) per deciliter
principal electrolytes in the extracellular water. However, (dL) of blood (mg/dL: a deciliter, or one tenth of a liter,
potassium (K) and phosphates (PO4) are the principal is the same as 100 mL). Since the concentrations of the
electrolytes in the cells. various electrolytes are small, they are expressed as
milliequivalents per liter, mEq/L.
Table 1: The Electrolyte concentration of body fluids
(mEq/L)* These values are shown in Table 1. Notice that the
concentration of the cations is the same as the
Solution Extracellular Intracellular
concentration of the anions in blood, serum or plasma
Fluid Fluid
when expressed in terms of milliequivalents. In other
Cations words, each liter of blood has 153 mEq of cations (Na, K,
Sodium 142 10 Ca, Mg) and 153 mEq of anions (bicarbonate, chloride,
sulphate, phosphate, organic acids and protein anions).
Potassium 4.5 150
Magnesium 2 40 The relationship between milliequivalents and
Calcium 4.5 milligrams
Total 153 200 The relationship between milliequivalents and
Anions milligrams can be expressed as follows : The weight of a
salt in milligrams can be converted into milliequivalents
Chloride 102
by dividing its weight in milligrams by its molecular
Phosphates 2 120 weight, and multiplying by the valence.
Sulphates 4 30
Example:
Bicarbonate 27 10
Protein 16 40 1 g of NaCl = 1000/58.5 mEq of NaCl = 17.1 mEq

Organic acids 5 Na (at. wt. 23; valence 1) Cl (at. wt. 35.5; valence 1)
Na + Cl = 58.5
Total 153 200
*Adapted from Wilson, RF. Critial Care Manual: Applied Physiology Since the electrolyte concentration in blood and serum
and Principles of Therapy, ed 2. F.A. Davis, Philadephia, 1992, p are usually expressed in mEq/L, the following formulas
656. can be used to convert mg/dL into mEq/L, or vice versa:
It should be noted that the electrolyte concentration mg/dL x 10 x valence
of the plasma (or serum) is slightly different from the mEq/L =
electrolyte concentration in the interstitial portion of Atomic weight
the extracellular water. The major difference is that
mEq/L x 10 x atomic weight
the protein concentration of the plasma is greater that mg/dL =
that of the interstitial portion of the extracellular water. 10 x valence

Electrolyte and Acid-Base Disturbances l APICON 2009 l Jan. 29 - Feb. 1, 2009 5


The activity constant of ions However, measurement of the anion gap (see below) is
a simpler way of describing an accumulation of abnormal
Measurement of the concentration of an ion or
anions.
electrolyte in the blood does not necessarily indicate
the quantity is completely available for chemical
reactions, because a portion may be bound to proteins, The anion gap
water, or other ions. A familiar example is the serum The anion gap (or delta) also describes the residual or
calcium concentration, which is partly bound to serum unmeasured anions.
albumin, and is partly ionized. A similar situation
exists with other ions, as Dahms and others have Since the concentration of anions such as phosphates,
shown. The ionized and non-ionized percentages of sulphates, and organic acid and protein anions is not
these ions in the blood can both be measured. ordinarily measured, the sum of the measured cations
However, the clinical importance of this information (Na, K, Ca) will be greater than the sum of the measured
is still not known for most ions. anions (HCO3 and Cl). This difference is known as the
anion gap (or delta).
Cation-anion balance The following simple formula can be used to determine
Table 1 shown that the sum of the cations in serum or if an abnormal gap is present:
plasma equals 153 mEq/L and the sum of the anions Anion gap = (Na + K) (HCO3 + Cl).
also equals 153 mEq/L.
Normally, the anion gap is 16 mEq/L or less. (An anion
This electrical equivalence of cations and anions in gap of less than 9 mEq/L is extremely unlikely, and is
serum or plasma is maintained regardless of whether probably the result of a laboratory error).
the sum of the cations (and anions) is greater or less
than 153 mEq/L. It may be greater, for example, when An alternate formula for measuring the anion gap is:
water loss occurs. It is often less that 153 mEq/L when Anion gap = Na (HCO3 + Cl)
electrolytes are lost from the body, as in vomiting or
When this formula is used, the average, normal value
diarrhea.
for the anion gap is 12 mEq/L, with a range of 8 to 16
All the cations can be determined clinically, although it mEq/L.
is usual to determine only sodium, potassium, and
calcium routinely. Chloride is the only anion that is Example:
routinely determined in studying electrolyte
disturbances. The bicarbonate concentration can be Na 142 K4 HCO3 27 Cl 103 mEq/L
calculated from the CO 2 content. Under ordinary Anion Gap = (Na + K) (HCO3 + Cl).
conditions, it is usually between 0.6 and 1.8 mEq/L less (142 +4 ) (27 + 103)
than the CO2 content. However, for purpose of simplicity
one can calculate HCO3 concentration by subtracting 1 146 130 = 16 mEq/L
mEq/L from the CO2 content. This is normal
The sum of phosphates, sulphates, organic acid anions,
Increased anion gap
and proteins is described by the symbol R.
1. Increased unmeasured anions, such as the
The significance of anions labeled R accumulation of lactic acid in lactic acidosis;
aceto-acetic acid in diabetic ketoacidosis; organic
The term R (residual ions) was used to describe o r ke t o a c i d s c a u s e d b y t h e i n g e s t i o n o f
unmeasured ions, such as plasma proteins and inorganic salicylates, paraldehyde, and ethylene glycol;
and organic acid anions in the plasma, which may be organic acids in hyperosmolar hyperglycaemic
present normally or may accumulate in some patients non-ketoacidotic diabetic coma; phosphoric,
with metabolic acidosis. sulphuric, and organic acids in azotaemia; formic
R is calculated as follow: acid in methanol (methyl alcohol) poisoning;
ketogutaric acid in hepatic failure; other anions,
R = Na + K + 6.5 (the sum of Ca and Mg) (HCO3 + Cl).
including high doses of (sodium) penicillin, and
Normally, the value of R is 22 mEq/L or less. (sodium) carbenicillin.

6 Electrolyte and Acid-Base Disturbances l APICON 2009 l Jan. 29 - Feb. 1, 2009


2. Decreased unmeasured cations, for example, concentration to be reduced without necessarily
potassium, in hypokalaemia (if the alternate formula affecting the sodium or chloride concentration.
above is used); calcium in hypocalcaemia;
magnesium in hypomagnesaemia. Osmotic Pressure
3. Metabolic alkalosis. The plasma proteins give up H A solute is a substance, such as sodium chloride,
ions and increase their net negative charge. As a potassium phosphate, glucose, or protein, which can
result, the measured anion concentration of the dissolve in a solvent, such as water, to make a solution.
plasma is apparently low. In addition, alkalosis is
usually associated with a decreased extracellular. The measure of the osmotic pressure of the salt solution
is dependent upon the number of particles or ions of
(The total anion value must equal the total cation the salt in a given volume of solution. In other words, a
value of the plasma. When alkalosis develops, the more concentrated salt solution would have a greater
anion value of the proteins increases. However, the osmotic pressure, and a less concentrated salt solution
anion value of the proteins is not measured would have a lesser osmotic pressure.
Therefore, there is a spurious decrease of the
measured anions, and an apparent increase in the Osmotic pressure is measured in terms of osmoles (Osm)
anion gap). or milliosmoles (mOsm). An osmole (or a milliosmole)
of a substance such as glucose, which does not dissociate
4. Increased unmeasured anions, such as increased into ions, is the same as a mole (or a millimole).
phosphate, or sulphate ions, or due to treatment with However, a mole of a salt such as sodium chloride, which
intravenous solutions containing lactate, citrate or dissociates almost completely into sodium and chloride
acetate ions. ions, equals 2 osmoles.
5. Laboratory errors, with falsely high serum Na or A mole of a salt such as sodium bicarbonate, which
falsely low serum Cl or HCO3 values. dissociates into sodium (Na) and bicarbonate (HCO3)
ions, also equals 2 osmoles. A mole of a more complex
The relation of serum sodium, bicarbonate and chloride
salt such as Na2HPO4, which dissociates into two Na
concentrations
and one HPO4 ions, equals 3 osmoles. The total osmotic
There is another relation between the cations and anions pressure of a solution therefore is calculated from the
that can be used, namely, the relation between serum sum of all the ions in solution.
sodium concentration and the sum of the serum
In the human and in animals, it is assumed that the
bicarbonate and chloride ions. In Table 1, it will be noted
osmotic pressures of the extracellular water and of the
that the most important cation is sodium, and that the
cells are the same. Water will flow from a region of low
most important anions are bicarbonate and chloride. In
to a region of higher osmotic pressure.
most cases, the following relations between these three
ions exist: In the human, the osmotic pressure of the extracellular
water and of the cells is 310 mOsm/L. The significance
Na = HCO3 + Cl + 12 mEq/L
of this is the following: In the extracellular fluid, sodium
Example: concentration is 142 mEq/L and the total cation
concentration is 155 mEq/L. Practically all the osmotic
Na 142 HCO3 27 Cl 103 mEq/L pressure of the extracellular water is a result of
142 = 27 + 103 + 12 monovalent salts, which ionize into two ions each.
Therefore, the osmotic pressure of the extracellular
In other words, if the bicarbonate and chloride ion water is twice the cation (or anion) concentration, or 2
concentration in serum or plasma are known, the x 155 = 310 mOsm/L. We can make the following
concentration of sodium can be determined from these assumption: Sodium is the chief cation of the
values, using the above rule. extracellular water; therefore osmotic pressure should
vary proportionately with the sodium concentration of
The major exception to this rule occurs in patients with
the serum or plasma. In other words, we should be able
a metabolic acidosis who show an abnormal anion gap,
to use the serum sodium concentration as measure of
because the abnormal anions are not ordinarily
the osmotic pressure of the extracellular water.
measured (see above). This increased anion
Unfortunately, this relationship is valid only in normal
concentration in the plasma causes the bicarbonate
subjects.

Electrolyte and Acid-Base Disturbances l APICON 2009 l Jan. 29 - Feb. 1, 2009 7


The difference between osmotic pressure and oncotic
pressure
The term osmotic pressure should be differentiated
from the term oncotic pressure, or colloid osmotic
pressure. The osmotic pressure of a solution varies with
the number of molecules in a solution, as was pointed
out above. When the molecular weight of a substance is
low, there will be more molecules of the substance per
unit weight, and the osmotic pressure will be greater.
For example, a substance such as mannitol, which has
a low molecular weight, will increase the osmotic
pressure when given intravenously. However, a
substance such as human albumin, or dextran, which
has high molecular weight around 80,000, will not
increase the osmotic pressure greatly. However, such a
substance exerts an oncotic pressure (colloid osmotic
pressure) because it is confined by a membrane (the
vascular system) to which it is relatively impermeable.
Therefore, an infusion of albumin or dextran will greatly
increase the oncotic pressure of the blood and will
prevent the loss of excessive fluid from the capillaries.
However, it will have an almost negligible effect on the
Fig. 1: Diagram showing the effect of osmotic pressure.
osmotic pressure of the blood. Oncotic pressure is
measured in terms of pressure units (mm Hg).
skin turgor and dry mucous membranes are poor
markers of decreased interstitial fluid. Signs of
Factors regulating the osmotic pressure and the volume
intravascular volume contraction include decreased
of the extracellular water
jugular venous pressure, postural hypotension, and
We have just pointed out that a primary change of postural tachycardia. Mild degrees of volume depletion
osmotic pressure, either in the extracellular water or are often not clinically detectable. Weight loss can help
in the cells, is associated with a shift of water into or estimate the magnitude of the volume deficit. Larger
out of the extracellular water, and consequently is fluid losses often present as hypovolaemic shock,
associated with a change in the volume of the heralded by hypotension, tachycardia, peripheral
extracellular water. vasoconstriction, and hypoperfusion-cyanosis, cold and
In the maintenance of life, the body has homeostatic or clammy extremities, oliguria, and altered mental status.
regulatory mechanisms that help to maintain the osmotic A thorough history and physical examination are
pressure and the volume of the extracellular water within generally sufficient to determine the presence and cause
physiological limits. The osmotic pressure of the of ECF volume contraction. Laboratory data confirm
extracellular water is controlled by the posterior and support the clinical diagnosis. Measurement of the
pituitary antidiuretic harmone and volume of fractional excretion of Na+ and BUN-creatinine ratio
extracellular fluid partly by the adrenal cortical hormone, may provide additional diagnostic information. There
aldosterone. may be a relative elevation in haematocrit
(haemoconcentration) and plasma albumin
concentration.
Hypovolaemia
Manifestations Aetiology
Symptoms are usually non-specific and secondary to Extracellular fluid (ECF) volume depletion reflects a
electrolyte imbalances and tissue hypoperfusion. These deficit in total body Na+ content as a result of renal or
include,thirst, fatigue, weakness, muscle cramps, and extrarenal losses that exceed Na+ intake. Renal losses
postural dizziness. More severe degrees of volume may be secondary to diuretics (pharmacological or
contraction can lead to syncope and coma. Diminished osmotic), interstitial renal disease (Na+ wasting), or

8 Electrolyte and Acid-Base Disturbances l APICON 2009 l Jan. 29 - Feb. 1, 2009


mineralocorticoid deficiency. Excessive renal losses of the post-operative state, and intubated patients in
Na+ and water may also occur during the diuretic phase the ICU. Rarely, impaired thirst may be caused by
of ATN and after the relief of bilateral urinary tract primary hypodipsia, as a result of damage to the
obstruction. Non-renal causes of hypovolaemia include hypothalamic osmoreceptors that control thirst.
fluid loss from the GI tract (vomiting, nasogastric Primary hypodipsia may be caused by a variety of
suction, fistula drainage, diarrhoea), skin and pathologic changes, including granulomatous
respiratory losses, third-space accumulations (burns, disease, vascular occlusion, and tumours.
pancreatitis, peritonitis), and haemorrhage.
B. Hypernatraemia due to water loss accounts for the
majority of cases of hypernatraemia. Because water
Treatment is distributed between the ICF and the ECF in a 2:1
The therapeutic goal is to restore normovolaemia with ratio, a given amount of solute-free water loss results
fluid similar in composition to that which was lost, as in the same percentage change but, quantitatively, a
well as to replace ongoing losses. Mild volume twofold greater absolute reduction in the ICF
contraction can usually be corrected via the oral route. compartment than the ECF compartment.
More severe cases of hypovolaemia require IV therapy.
1. Non-renal water loss may be due to evaporation
Patients with significant haemorrhage, anaemia. or third-
from the skin and respiratory tract (insensible
spacing may require blood transfusion or colloid-
losses) or loss from the GI tract. Insensible losses
containing solutions (albumin, dextran). Isotonic or
are increased with fever, exercise, heat exposure,
normal saline (0.9% NaCI or 154 mEq/L Na+) is the
severe burns, and in mechanically ventilated
solution of choice in normonatraemic and mildly
patients. Diarrhoea is the most common GI
hyponatraemic individuals and should also be used
cause of hypernatraemia. Specifically, osmotic
initially in patients with hypotension or shock. Severe
diarrhoeas (induced by lactulose, sorbitol, or
hyponatraemia may require hypertonic saline (3.0%
malabsorption of carbohydrate) and viral
NaCI or 513 mEq/L Na+). Hypokalaemia may be present
gastroenteritis result in water loss exceeding that
initially or may ensue as a result of increased urinary
of Na+ and K+.
K + excretion and should be corrected by adding
appropriate amounts of KCI to replacement solutions. 2. Renal water loss is the most common cause of
Finally, the appropriate management of hypovolaemia hypernatraemia and results from either osmotic
must include correction of the underlying cause. diuresis or diabetes insipidus. The most frequent
cause of an osmotic diuresis is hyperglyceamia
and glycosuria in poorly controlled diabetes
Hypernatraemia
mellitus. IV administration of mannitol and
Hypernatraemia is defined as a plasma [Na+] of greater increased production of urea (high-protein diet)
than 145 mEq/L and presents a state of hyperosmolality. can also result in an osmotic diuresis.
Maintenance of osmotic equilibrium in hypernatremia Hypernatraemia secondary to non-osmotic
results in ICF volume contraction and cerebral cell urinary water loss is usually caused by (1)
shrinkage. Hypernatraemia may be caused by a primary central diabetes insipidus (CDI) characterized
Na+ gain or water deficit. The two components of an by impaired vasopressin secretion or (2)
appropriate response to hypernatraemia are increased nephrogenic diabetes insipidus (NDI) that
water intake stimulated by thirst and the excretion of results from resistance to the actions of
the minimum volume of maximally concentrated urine, vasopressin. The most common cause of CDI is
reflecting vasopressin secretion in response to an destruction of the neurohypophysis as a result
osmotic stimulus. of trauma, neurosurgery, granulomatous dis-
ease, neoplasms, vascular accidents, or infection.
Aetiology In many cases, CDI is idiopathic and may
occasionally be hereditary. NDI may either be
A. Impaired thirst: The degree of hyperosmolality is inherited or acquired. The latter can be further
typically mild unless the thirst mechanism is subdivided into disorders associated with renal
abnormal or access to water is limited. The latter medullary disease or with impaired vasopressin
occurs in infants, the physically handicapped, action. The causes of sporadic NDI are
patients with impaired mental status, individuals in numerous and include drugs (especially lithium),

Electrolyte and Acid-Base Disturbances l APICON 2009 l Jan. 29 - Feb. 1, 2009 9


hypercalcaemia, hypokalaemia, and conditions 100 mEq/L). Many causes of hypernatraemia are
that impair medullary hypertonicity (e.g., associated with polyuria and a submaximal urine
papillary necrosis or osmotic diuresis). osmolality. Calculation of the total daily solute
excretion (24-hr urine volume x urine osmolality)
C. Hypernatraemia due to Na+ gain occurs infrequently.
is helpful in determining the basis of polyuria. To
This is most commonly seen in patients with diabetic
maintain a steady state, total solute excretion must
ketoacidosis (DKA) and an osmotic diuresis (urine
equal solute production. As mentioned previously,
Na +< 50 mEq/L) treated with isotonic saline.
a daily solute excretion in excess of 900 mOsm
Inadvertent administration of hypertonic NaCl or
defines an osmotic diuresis. This can be confirmed
NaHCO3 or replacing sugar with salt in infant formula
by measuring urine glucose and urea.
can also lead to hypernatraemia.
2. CDI and NDI generally present with polyuria and
D. Transcellular water shift from ECF to ICF occurs in
hypotonic urine (urine osmolality <250 mOsm/kg).
rare circumstances (e.g., secondary to seizures or
The degree of hypernatraemia is usually mild unless
rhabdomyolysis). Hypernatraemia is accompanied
the patient has an associated thirst abnormality. The
by ECF volume contraction with no change in body
clinical history, physical examination, and pertinent
weight.
laboratory data can often rule out causes of acquired
NDI. CDI and NDI can be distinguished by
Manifestations administering the vasopressin analog DDAVP (10
The major symptoms of hypernatraemia are neurologic microgram intranasally) after careful water
and include altered mental status, weakness, restriction. The urine osmolality should increase by
neuromuscular irritability, focal neurologic deficits, and at least 50% in CDI and does not change in NDI.
occasionally coma or seizures. Patients may also The diagnosis is sometimes difficult due to partial
complain of polyuria or thirst. For unknown reasons, defects in vasopressin secretion and action.
patients with polydipsia from CDI tend to prefer ice-
cold water. The signs and symptoms of volume depletion Treatment
are often present in patients with a history of excessive The therapeutic goals are (1) to stop ongoing water loss
sweating, diarrhoea, or osmotic diuresis. As with and (2) to correct the water deficit. The ECF volume
hyponatraemia, the severity of the clinical should be restored in hypovolaemic patients. The
manifestations is related to the acuity and magnitude of quantity of water required to correct the deficit can be
the rise in plasma [Na+]. Chronic hypernatraemia is calculated from the following equation:
generally less symptomatic as a result of adaptive
mechanisms designed to defend cell volume. (plasma [Na+] 140)
Water deficit = x total body water (in liters)
140
Diagnosis
1. The rate of correction: Rapid correction of
A complete history and physical examination often hypernatraemia is potentially dangerous due to a
provide clues as to the underlying cause of rapid shift of water into brain cells, increasing the
hypernatraemia. The history should include a list of risk of seizures or permanent neurologic damage.
current and recent medications, and the physical Therefore, the water deficit should be corrected
examination is incomplete without a thorough mental slowly over at least 48-72 hours. When calculating
status and neurologic assessment. the rate of water replacement, ongoing losses should
1. Assessment of urine volume and osmolality is be taken into account, and the plasma sodium should
essential in the evaluation of hyperosmolality. The be lowered by 0.5 mEq/L/hour and by no more than
appropriate renal response to hypernatraemia is 12mEq/L over the first 24 hours. The safest route
excretion of the minimum volume (500 mL/day) of of administration of water is by mouth or via a
maximally concentrated urine (urine osmolality> nasogastric tube. Alternatively, half-isotonic saline
800 mOsm/kg). These findings suggest extrarenal can be given IV.
or remote renal water loss or administration of 2. CDl: The appropriate treatment of CDI consists of
hypertonic Na+ salt solutions. A primary Na+ excess administering DDAVP intranasally.
can be confirmed by the presence of ECF volume
expansion and natriuresis (urine [Na+] usually > 3. NDI: The concentrating defect in NDI may be

10 Electrolyte and Acid-Base Disturbances l APICON 2009 l Jan. 29 - Feb. 1, 2009


reversible by treating the underlying disorder or Oliguric acute and chronic renal failure may be
eliminating the offending drug. Symptomatic associated with hyponatraemia if water intake
polyuria caused by NDI can be treated with a low exceeds the kidneys limited ability to excrete
Na+ diet and thiazide diuretics. This results in mild equivalent volumes.
volume depletion, enhanced proximal reabsorption
of salt and water, and decreased delivery to the site 3. Hyponatraemia associated with a normal ECF
of action of vasopressin, the collecting duct. NSAlDs volume
potentiate vasopressin action and thereby increase a. The syndrome of inappropriate antidiuretic
urine osmolality and decrease urine volume. hormone secretion (SIADH) is the most
common cause of normovolaemic
Hyponatraemia hyponatraemia. This disorder is caused by
the non-physiologic release of vasopressin
This is a condition in which the sodium concentration from the posterior pituitary or an ectopic
in the plasma is low (hypo means below in Greek; in source, resulting in impaired renal free water
this case, below 135 mmol/L). excretion. Common causes of SIADH
include neuropsychiatric disorders,
Aetiology pulmonary diseases, and malignant tumours.
A. Hyponatraemia with a low plasma osmolality SIADH is characterized by (1) hypotonic
hyponatraemia, (2) an inappropriately
Most causes of hyponatraemia are associated with concentrated urine (urine osmolality >100
a low plasma osmolality (high ICF volume). In mOsm/kg), (3) euvolaemia, and (4) normal
general, hypotonic hyponatraemia is caused either renal, adrenal, and thyroid function.
by a primary water gain or Na+ loss. The ECF
volume, reflecting total body Na+ content, may be b. Glucocorticoid deficiency and
decreased, normal, or increased in hyponatraemia. hypothyroidism may present with
hyponatraemia and should not be confused
1. Hyponatraemia associated with ECF volume with SIADH. Although decreased
depletion may result from renal or non-renal mineralocorticoids may contribute to the
causes of net Na+ loss. A decreased effective hyponatremia seen in Addisons disease, it
arterial volume stimulates thirst. It also is the cortisol deficiency that leads to hyper-
stimulates vasopressin release from the posterior secretion of vasopressin directly (co-secreted
pituitary gland, which impairs the capacity to with corticotropin-releasing factor) and
excrete a dilute urine. Hyponatraemia develops indirectly (secondary to volume depletion).
as a consequence of electrolyte-free water The mechanisms by which hypothyroidism
retention. Furthermore, certain causes of leads to hyponatraemia include decreased
hypovolaemic hyponatraemia (e.g., diuretics or cardiac output and glomerular filtration rate
vomiting) may be associated with a large K+ (GFR) and increased vasopressin secretion
deficit, resulting in transcellular ion exchange in response to haemodynamic stimuli.
(K + exits and Na + enters cells), which
c. Pharmacologic agents may cause
contributes to hyponatraemia.
hyponatraemia by one of at least three
2. Hyponatraemia associated with ECF volume mechanisms: (1) stimulation of vasopressin
excess is usually a consequence of oedematous release (e.g., nicotine, carbamazepine,
states, such as CHF, hepatic cirrhosis, and tricyclic antidepressants, antipsychotic
nephrotic syndrome. These disorders all have in agents, antineoplastic drugs, narcotics), (2)
common a decreased effective circulating potentiation of antidiuretic action of
volume, leading to increased thirst and vasopressin [e.g., chlorpropamide,
vasopressin levels. The increase in total body methylxanthines, non-steroidal anti-
Na+ is exceeded by the rise in total body water. inflammatory drugs (NSAIDs)], or (3)
The degree of hyponatraemia often correlates vasopressin analogs [e.g., oxytocin,
with the severity of the underlying condition and desmopressin acetate (DDAVP)].
is therefore an important prognostic factor. d. Physical and emotional stress are often

Electrolyte and Acid-Base Disturbances l APICON 2009 l Jan. 29 - Feb. 1, 2009 11


associated with vasopressin release, possibly artificially lowers the Na+ measured per liter of
secondary to nausea and/or hypotension plasma (except when Na + sensitive glass
associated with stress-induced vasovagal electrodes are used). The plasma osmolality and
reactions. the Na+ measured per liter of plasma water
remain normal.
e. Acute hypoxia or hypercapnia also
stimulates vasopressin secretion. 2. Hyponatraemia associated with a hyperosmolar
state is usually caused by an increase in the
f. Psychogenic polydipsia refers to a condition
concentration of a solute that is largely restricted
of compulsive water consumption that may
to the ECF compartment. The resulting osmotic
overwhelm the normally large renal
gradient leads to water shift from the ICF to the
excretory capacity of 12 L/day. These
ECF, and hyponatraemia ensues. Hypertonic
patients often have psychiatric illnesses and
hyponatraemia is usually caused by
may be taking medications, such as
hyperglycaemia or occasionally IV
phenothiazines, that enhance the sensation
administration of mannitol. Quantitatively, the
of thirst by causing a dry mouth.
plasma [Na+] falls by 1.4 mEq/L for every 100
g. Beer potomania is similar to psychogenic mg/dl rise in the plasma glucose concentration.
polydipsia but with an associated lower renal
excretory capacity of water. Urine can be Manifestations
maximally diluted to 50 mOsm/L. The low-
solute and low-protein diet seen with The clinical features of acute hyponatraemia are related
excessive beer intake may only result in the to osmotic water shift leading to increased ICF volume,
generation of 200-250 mOsm/day (600-900 specifically cerebral oedema. Therefore, the symptoms
mOsm/day is normal). Thus, only 4-5 L/day are primarily neurologic, and their severity is dependent
of urine can be generated. Beer drunk in on the rapidity of onset and absolute decrease in plasma
excess of this capacity results in [Na+]. Patients may be asymptomatic or may complain
hyponatraemia. A similar state, often of nausea and malaise. As, the plasma [Na+] falls, the
referred to as the tea-and-toast diet, has been symptoms progress to include headache, lethargy,
observed in malnourished elderly patients confusion, and obtundation. Stupor, seizures, and coma
who maintain fluid intake without an do not usually occur unless the plasma [Na+] falls
adequate diet. acutely below 120 mEq/L. In chronic hyponatremia,
adaptive mechanisms designed to defend cell volume
h. Cerebral salt wasting is a controversial and occur and tend to minimize the increase in ICF volume
poorly understood syndrome that has been and its symptoms.
associated with neurosurgery and CNS
trauma. It is purportedly distinguished from Diagnosis
SIADH by a negative sodium balance and
intravascular volume depletion after a CNS The underlying cause of hyponatraemia can often be
injury. The contorversy centers on the tenet ascertained from an accurate history and physical
that the underlying hyponatremia is best examination, including an assessment of ECF volume
treated with hydration and saline, and not status and effective circulating arterial volume. Three
with fluid restriction. laboratory findings often provide useful information and
can narrow the differential diagnosis of hyponatraemia:
(1) the plasma osmolality, (2) the urine osmolality, and
B. Hyponatraemia with a normal or high plasma
(3) the urine [Na +] + [CI-].
osmolality
1. Pseudohyponatraemia is hyponatraemia 1. Plasma osmolality: Because ECF tonicity is
associated with a normal plasma osmolality. It determined primarily by the [Na+], most patients
occurs as a result of a decrease in the aqueous with hyponatraemia have a decreased plasma
phase of plasma. Plasma is 93% water, with the osmolality. If the plasma osmolality is not low,
remaining 7% consisting of plasma proteins and pseudohyponatraemia and hypertonic
lipids. Because Na+ ions are dissolved in plasma hyponatraemia must be ruled out.
water, increasing the non-aqueous phase 2. Urine osmolality and volume: The appropriate renal

12 Electrolyte and Acid-Base Disturbances l APICON 2009 l Jan. 29 - Feb. 1, 2009


response to hypoosmolality is to excrete the cirrhosis tends to reflect the severity of the
maximum volume of dilute urine that is, urine underlying disease and is usually asymptomatic.
osmolality and specific gravity of less than 100 Treatment should include restriction of Na+ and
mOsm/kg and 1.003 respectively. This occurs in water intake, correction of hypokalaemia, and
patients with primary polydipsia. If this is not promotion of water loss in excess of Na+. The latter
present, it suggests impaired free water excretion may require the use of loop diuretics with
due to the action of vasopressin on the kidney. The replacement of a proportion of the urinary Na+ loss
secretion of vasopressin may be a physiologic to ensure net free water excretion. Dietary water
response to haemodynamic stimuli, or it may be restriction should be less than the urine output.
inappropriate in the presence of hyponatraemia and Correction of K+ deficit may raise the plasma [Na+].
euvolaemia. The maximal urine output is a function
of the minimum urine osmolality achievable and the 3. The rate of correction of hyponatraemia depends
mandatory solute excretion. Metabolism of a normal on the absence or presence of neurologic
diet generates 600-900 mOsm/day, and the minimum dysfunction. This, in turn, is related to the rapidity
urine osmolality in humans is approximately 50 of onset and magnitude of the fall in plasma Na+.
mOsm/kg. Therefore, the maximum daily urine The risks of correcting hyponatremia too rapidly
output will be 12 L or more (600/50 = 12). A solute are ECF volume excess and the development of
excretion rate of greater than 900 mOsm/day is, by osmotic demyelination or central pontine
definition, an osmotic diuresis. A low-protein, low- myelinolysis. This disorder, in its most overt form
salt diet may yield as few as 100 mOsm/day, which is characterized by flaccid paralysis, dysarthria, and
translates into a maximal urine output of 2 L/day at dysphagia. The diagnosis is occasionally suspected
a minimum urine tonicity of 50 mOsm/kg. Moreover, clinically and can be confirmed by appropriate
net Na+ loss and ECF volume contraction lead to neuroimaging studies (CT scan or MRI). In
vasopressin release, further impairing free water addition to rapid or over-correction of
excretion. hyponatraemia, risk factors for osmotic
demyelination include hypokalaemia and
3. Urine Na+ concentration: Because Na+ is the major malnutrition, especially secondary to alcoholism.
ECF cation and is largely restricted to this
compartment, ECF volume contraction represents 4. Acute hyponatraemia tends to present with altered
a deficit in total body Na+ content. Therefore, volume mental status or seizures, or both, and requires more
depletion in patients with normal underlying renal rapid correction. Severe symptomatic
function results in enhanced tubule Na + hyponatraemia should be treated with hypertonic
reabsorption and a urine [Na+] of less than 20 mEq/ saline, and the plasma Na+ should be raised only by
L. The finding of a urine [Na+] of greater than 20 1-2 mEqL/hour and by no more than 8 mEq/L
mEq/L in hypovolaemic hyponatraemia implies during the first 24 hours. The quantity of Na+ that
diuretic therapy, hypoaldosteronism, or occasionally, is required to increase the plasma Na+ concentration
vomiting. by a given amount can be estimated by multiplying
the desired change in plasma Na+ by the total body
Treatment water (e.g., 5 mEq/L x 30 L = 150 mEq = 300 ml
3% NaCI). In asymptomatic patients, the plasma
The goals of therapy are threefold: (1) raise the plasma Na+ should be raised by no more than 0.3 mEq/L/
[Na+] (lowering the ICF volume) by restricting water hour and equal to or less than 8 mEq/L over the
intake and promoting water loss, (2) replace the Na+ first 24 hours.
and K+ deficit(s), and (3) correct the underlying disorder.
Mild asymptomatic hyponatraemia is generally of little 5. Water restriction in primary polydipsia and IV saline
clinical significance and requires no treatment. therapy in ECF volume-contracted patients may also
lead to overly rapid correction of hyponatraemia as
1. ECF volume contraction: Management of a result of vasopressin suppression and a brisk water
asymptomatic hyponatraemia should include Na+ diuresis. This can be prevented by administration
repletion, generally in the form of saline that is of water or use of a vasopressin analog to slow down
isotonic to the patient, to avoid rapid changes in the rate of free water excretion.
ICF volume.
6. The hyponatraemia of SIADH can be treated by
2. Oedematous states: Hyponatraemia in CHF and limiting the intake of water or promoting its

Electrolyte and Acid-Base Disturbances l APICON 2009 l Jan. 29 - Feb. 1, 2009 13


Fig. 2: Differential diagnosis of hyponatraemia.

14 Electrolyte and Acid-Base Disturbances l APICON 2009 l Jan. 29 - Feb. 1, 2009


excretion, or both. The standard first-line therapy is Leukaemia (mechanism uncertain)
water restriction. If this fails or if the patient is l GI losses
symptomatic, agents that enhance water excretion Vomiting or nasogastric suctioning
can be tried. Loop diuretics impair the ability to
Diarrhoea
excrete concentrated urine and, when combined
with Na+ replacement in the form of salt tablets, Enemas or laxative use
can enhance free water excretion. In SIADH, the Ileal loop
urine osmolality is relatively fixed. Therefore, the l Medication effects
maximum urine output is a direct function of the Diuretics (most common cause)
solute excretion rate, which can be increased by Beta-adrenergic agonists
dietary modification (high salt, high protein) or by Steroids
administering urea, leading to increased urine output Theophylline
and water excretion. Drugs that interfere with the
Aminoglycosides
collecting tubules ability to respond to vasopressin
include lithium and demeclocycline. These agents l Transcellular shift
are rarely used and should only be considered in Insulin
severe hyponatraemia that is unresponsive to more Alkalosis
conservative measures. l Malnutrition or decreased dietary intake, parenteral
nutrition
Hypokalaemia
Manifestations
Hypokalaemia refers to the condition in which the
concentration of potassium in the blood is low. The History
prefix hypo- means low (contrast with hyper-, meaning
high). Kal refers to kalium, the Neo-Latin for potassium, The history may be vague
and -emia means in the blood. Common symptoms include the following:
Normal serum potassium level is between 3.5 to 5.0 l Palpitations
mEq/L. At least 95% of the bodys potassium is found l Skeletal muscle weakness or cramping
inside cells, with the remainder in the blood. This l Paralysis, paraesthesias
concentration gradient is maintained principally by the l Constipation
Na+/K+-ATPase pump. l Nausea or vomiting
l Abdominal cramping
Pathophysiology l Polyuria, nocturia, or polydipsia
l Potassium is essential for many body functions, l Psychosis, delirium, or hallucinations
including muscle and nerve activity l Depression
l Decreased potassium levels in the extracellular space
will cause hyperpolarization of the resting Physical examination
membrane potential l Signs of ileus
l In certain conditions, this will make cells less l Hypotension
excitable. However, in the heart, it causes myocytes l Ventricular arrhythmias
to become hyperexcitable. l Cardiac arrest
l This delayed repolarization may promote reentrant l Bradycardia or tachycardia
arrythmias l Premature atrial or ventricular beats
l Hypoventilation, respiratory distress
Aetiologies l Respiratory failure
l Renal losses l Lethargy or other mental status changes
Renal tubular acidosis l Decreased muscle strength, fasciculations, or tetany
Hyperaldosteronism l Decreased tendon reflexes
Magnesium depletion l Cushingoid appearance (e.g., oedema)

Electrolyte and Acid-Base Disturbances l APICON 2009 l Jan. 29 - Feb. 1, 2009 15


Lab studies Treatment
l Serum potassium level <3.5 mEq/L (3.5 mmol/L)
Pre-hospital care
l BUN and creatinine level
l Be attentive to the ABCs.
l Glucose, magnesium, calcium, and/or phosphorus
l If the patient is severely bradycardic or manifesting
level if co-existent electrolyte disturbances are
cardiac arrhythmias, appropriate pharmacologic
suspected.
therapy or cardiac pacing should be considered.
l Consider digoxin level if the patient is on a digitalis
preparation; hypokalaemia can potentiate digitalis- Emergency department care
induced arrhythmias
l Patients in whom severe hypokalaemia is suspected
l Consider arterial blood gas (ABG). Alkalosis can should be placed on a cardiac monitor; establish
cause potassium to shift from extracellular to intravenous access and assess respiratory status.
intracellular space l Direct potassium replacement therapy by the
symptomatology and the potassium level. Begin
Imaging studies therapy after laboratory confirmation of the diagnosis.
l CT scan of the adrenal glands is indicated if l Usually, patients who have mild to moderate
mineralocorticoid excess is evident (rarely needed hypokalaemia (potassium of 2.5-3.5 mEq/L), are
emergently asymptomatic, or have only minor symptoms and
need only oral potassium replacement therapy. If
Electrocardiography (Figure 3) cardiac arrhythmias or significant symptoms are
l T-wave flattening or inverted T waves present, then more aggressive therapy is warranted.
This treatment is similar to the treatment of severe
l Prominent U wave that appears as QT prolongation
hypokalaemia.
l ST-segment depression
l If the potassium level is less than 2.5 mEq/L,
l Ventricular arrhythmias (e.g., premature ventricular intravenous potassium should be given. Admission
contractions [PVCs], torsade de pointes, ventricular or observation in emergency department is indicated;
fibrillation) replacement therapy takes more than a few hours.
l Atrial arrhythmias (e.g., premature atrial l Serum potassium level is difficult to replenish if serum
contractions [PACs], atrial fibrillation) magnesium level is also low. Look to replace both.

Fig. 3: ECG showing prominent U waves after T waves and a premature ventricular ectopic (interposed between two QRS complexes).

16 Electrolyte and Acid-Base Disturbances l APICON 2009 l Jan. 29 - Feb. 1, 2009


Medication potassium level.
Potassium Potassium depletion sufficient to
Complications
chloride cause 1 mEq/L drop in serum
potassium indicates loss of about 100- l Replacing potassium too quickly can cause a rapid
200 mEq of potassium from total body rise in the blood potassium level, leading to a relative
store. In the symptomatic patient with hyperkalaemia with subsequent cardiac
severe hypokalaemia, administer up to complications.
40 mEq/h of this IV preparation; l If hypokalaemia is not corrected easily with
maintain close follow-up; provide replacement therapy, search for other co-existent
continuous ECG monitoring, and metabolic abnormalities (e.g., hypomagnesaemia).
check serial potassium levels. Higher Hypokalaemia may be refractory to treatment until
dosages may increase risk of cardiac hypomagnesaemia is corrected.
complications. l Hypokalaemia can potentiate digitalis toxicity in
patients who are taking digoxin
Adult dose 10-20 mEq/h IV via peripheral or
central line Patient education
Pediatric dose 0.5-1 mEq/kg/dose over 1 h; not to l Diet modification is recommended for those patients
exceed adult maximum dose who are predisposed to hypokalaemia. Increase
intake of bananas, tomatoes, oranges, and peaches
Contraindications Hyperkalaemia; renal failure; because they are high in potassium.
conditions in which potassium is
retained; oliguria or azotaemia; crush Special concerns
syndrome; severe haemolytic reactions; l Do not overcorrect potassium in patients with
anuria; adrenocortical insufficiency periodic hypokalaemic paralysis. This condition is a
Precautions Do not infuse rapidly; high plasma transcellular maldistribution, not a true deficit.
concentrations of potassium may cause l Diuretic therapy, diarrhoea, and chronic laxative
death due to cardiac depression, abuse are the most common causes of hypokalaemia
arrhythmias, or arrest; plasma levels do in elderly patients.
not necessarily reflect tissue levels; l In patients with hypokalaemia and diabetic
monitor potassium replacement ketoacidosis, part of the serum potassium should be
therapy whenever possible by administered as potassium phosphate.
continuous or serial ECGs; when
concentration >40 mEq/L infused,
Hyperkalaemia
local pain and phlebitis may occur
Hyperkalaemia is defined as potassium level greater than
Follow-up 5.5mEq/L. Based on serum potassium concentration,
hyperkalaemia can be divided into mild, moderate or
Further inpatient care severe.
l Continue intravenous replacement of potassium as
Mild : 5.5 - 6.0 mEq/L
needed.
l Continue cardiac monitoring in severe hypokalaemia. Moderate : 6.1 - 7.0 mEq/L
l Repeat potassium level measurement every 1-3 hours. Severe : > 7 mEq/L
l Identify the aetiology of the hypokalaemia.
Etiologies of hyperkalaemia
Further outpatient care
1. Intercompartmental shift:
l Repeat potassium level in 2-3 days.
l Acidosis
Inpatient or outpatient medication l Hypertonicity
l Consider switching to potassium-sparing diuretic if l Rhabdomyolysis
diuretic therapy is needed. l Succinylcholine
l Take 40 mEq KCI daily for 2-3 days and repeat the l Digoxin intoxication

Electrolyte and Acid-Base Disturbances l APICON 2009 l Jan. 29 - Feb. 1, 2009 17


l Hyperkalaemic familial periodic paralysis 3. Potassium > 7 mEq/L produces wide p wave,
delayed AV conduction, prolonged PR interval
2. Decreased renal potassium excretion
4. Potassium >7.5 mEq/L leads to cessation of atrial
l Renal failure
contraction and QRS widens resembling a sine wave.
l Decreased mineralocorticoid activity Death occurs due to ventricular fibrillation or
l Defective tubular secretion ( renal tubular cardiac arrest.
acidosis 2 & 4)
l Drugs (Spironolactone, ACE inhibitors, Management of hyperkalaemia
cyclosporine, NSAIDs)
Diagnosis depends upon lab. studies, clinical features
3. Increased potassium intake and ECG changes.
l Salt substitute 1. Calcium gluconate
4. Pseudohyperkalaemia Intravenous 10 ml calcium gluconate 10% over 2-5
l Haemolysis minutes.
l Thrombocytosis Onset of action 5 minutes
l Leucocytosis Duration of action 30-60 minutes
l Improper venepuncture technique (ischaemic Repeat dose in 3-5 minutes
blood drawn due to prolonged tourniquet Mechanism of action Potassium antagonism and
application) membrane stabilization
2. Insulin-dextrose infusion
Clinical manifestations Mechanism of action Intracellular shift of
potassium
Skeletal and cardiac muscle
Onset of action 20-30 minutes
Skeletal muscle weakness is seen when potassium level Dose 10 to 20 units regular insuln and 25-50 g
is greater than 8 mEq/L. It occurs due to spontaneous glucose
sustained depolarization and inactivation of sodium
channels on muscle membrane resulting in ascending 3. Intravenous infusion of sodium bicarbonate
paralysis. Cardiac manifestations occur when potassium Mechansim of action Intracellular shift of
level is above 7 mEq/L. This results in delayed potassium
depolarization of cardiac muscles. Dose 8.4% solution in children and adults and
Various signs and symptoms of hyperkalaemia include 4.2% in infants. Give 1 mEq/Kg slow intravenously
the following: as infusion.
Precaution Bicarbonate can bind with calcium;
Generalised fatigue
therefore, administer bicarbonate 30-60 minutes
Weakness after calcium gluconate.
Paraesthesias
4. Beta 2 agonist (salbutamol)
Paralysis (Decreased motor power, diminished deep
Promotes cellular uptake of potassium via cyclic
tendon reflexes)
GMP receptor cascade.
Palpitations, extrasystoles
Dose 5-20 mg mixed with 5 mL of saline via a high
Bradycardia, junctional rhythm, heart blocks
flow nebulizer over 20 minutes.
Hypoventilation (rare)
5. Diuretics
Signs of trauma(leading to hyperkalaemia)
Furosemide 20-40 mg intravenously
Signs of renal failure (oedema, skin changes)
Onset of action 60 minutes
Death occurs from cardiac arrest
6. Sodium polystyrene sulfonate (Kayexalate)
ECG features Dose 10-15 g mixed with 100 mL of 20% sorbitol
1. Peaking of T waves and shortened QT interval orally or per rectally
2. Potassium > 6 mEq/L produces wide QRS (Bundle 7. Dialysis is the most effective method when other
branch block) methods fail.

18 Electrolyte and Acid-Base Disturbances l APICON 2009 l Jan. 29 - Feb. 1, 2009


2. Acid-Base Disturbances
Praveen Aggarwal

Disturbances in acid-base balance are commonly maintaining the buffering capacity of the
encountered in the intensive care units and emergency extracellular fluid (ECF). Other minor buffers in
departments. These disorders may be life-threatening ECF include phosphates and proteins.
in themselves without regard to the underlying
2. The second physiological buffers are the
conditions causing them. Appropriate diagnosis and
intracellular proteins, of which, haemoglobin
management of these disorders in an acutely ill patient
is most important. It can buffer large amounts
require accurate and timely interpretation of specific
of H+ ions without disturbing the pH. In the
data. Physicians must be able to interpret the numbers
red cells, carbon dioxide combines with water
rapidly and accurately. In the present topic, a brief of
to form carbonic acid which dissociates to
physiological mechanisms by which acid-base balance
produce H + ions that are buffered by
is maintained in the body will be discussed. This will be
haemoglobin. Other buffers are intracellular
followed by various definitions used in acid-base
proteins and phosphates.
disorders, and detailed description of acid-base disorders
and their treatments. Finally, step-by-step approach to 3. Bone contains large amount of bicarbonate and
analyze acid-base disturbance will be discussed which can buffer parts of acute acid load. An acid load
will be followed by a case to illustrate various principles is associated with uptake of some of the excess
of acid-base interpretation. H+ ions by bone. This can occur in exchange for
surface Na+ and K+, and by dissolution of bone
mineral, resulting in the release of buffer
Normal Physiology of Acid-Base Balance compounds such as NaHCO 3 and KHCO 3
Acids are produced continuously during normal initially and then CaCO3 and CaHPO4 into
metabolism of body. About 20,000 mmol of carbonic extracellular fluid. The role of bone buffers may
acid and 80 mEq of non-volatile acids are produced daily. be even greater in the presence of a chronic acid
Despite this, the pH of extracellular fluid is maintained retention, such as seen in patients with chronic
between 7.36-7.44. The defense against fluctuations in renal failure.
pH is provided by three physiological buffer systems,
respiratory mechanics and renal mechanics. B. Pulmonary mechanisms
The principal volatile acid of metabolism is carbon
A. Physiological buffers dioxide which is equivalent to potential carbonic
acid. The normal concentration of carbon dioxide
The body buffers which are primarily weak acids,
in the body is maintained around 1.2 mmol/L by
are able to take up or release H+ so that changes in
the lungs. At this concentration, the pulmonary
the free H+ concentration are minimized. In the
excretion equals the metabolic production of carbon
body, three type of physiological buffers exist. These
dioxide.
are as follows:
1. The major physiological buffer is the bicarbonate- C. Renal mechanisms
carbonic acid system. Bicarbonate is converted
into water and carbon dioxide whenever H+ ions Kidneys reabsorb the filtered bicarbonate and also
are added. Carbon dioxide thus liberated is regenerate fresh bicarbonate. Bicarbonate is
excreted by the lungs. As the total buffering reabsorbed both in the proximal (75%) and distal
capacity of this system is about 15 mEq/L, at segments (25%) by secretion of protons into the
normal rate of production of non-volatile acids, tubular fluid. For each molecule of bicarbonate
the buffers will be depleted within a period of 15- filtered, one molecule is added to the blood by this
20 days. However, kidneys have the ability to mechanism (Figure 1).
regenerate bicarbonate and therefore help in New bicarbonate is regenerated by secretion of

Electrolyte and Acid-Base Disturbances l APICON 2009 l Jan. 29 - Feb. 1, 2009 19


Fig. 3: Formation of NH4 in Renal Tubules.

extracellular volume enhances renal bicarbonate


Fig. 1: Renal Reabsorption of Filtered Bicarbonate reabsorption.
3. Aldosterone levels: Hyperaldosteronism stimulates
bicarbonate reabsorption by the kidneys and can
lead to alkalosis.
4. Body potassium stores: Severe potassium depletion
produces increased hydrogen ion secretion and
therefore produces alkalosis due to increased
bicarbonate reabsorption.

Evaluation of Acid-Base Status


Evaluation of acid-base status of a patient is dependent
upon changes in the major buffer i.e., the bicarbonate-
carbonic acid system. The relationship between
bicarbonate and carbonic acid is described by
Handerson-Hasselbach equation:
Fig. 2: Renal Tubular Titrable Acidity by HPO4.
Bicarbonate
pH = pKa x Log
Carbonic acid
protons onto urinary buffers. About one-third is
titrated to phosphate while the remaining is secreted pKa is known as the dissociation constant and for
as ammonium. (Figure 2 and Figure 3). carbonic acid-bicarbonate system, it is 6.1. Carbonic acid
can be expressed as dissolved carbon dioxide and equals
The net acid excretion by kidneys = urinary NH4 plus x pCO2 where represents solubility coefficient and
urinary titrable acid (H2PO4) minus urinary HCO3. equals 0.031 mmol/L/mmHg of CO2. At a pCO2 of 40
Urinary bicarbonate is almost nil. mmHg, carbonic acid will be 1.2 mmol/L. The arterial
The rate of proton secretion by the kidneys is influenced blood gas analyzers measure the pH and pCO2 and
by a number of factors: bicarbonate concentration is calculated using the above-
mentioned formula. By modifying the equation stated
1. Carbon dioxide tension: Bicarbonate reabsorption above, a more practical equation can be derived:
is directly related to the ECF carbon dioxide tension.
Hypercaponea stimulates and hypocaponea inhibits H+ in nmol/L = 24 x pCO2/HCO3
renal bicarbonate reabsorption. At a pH of 7.4, the H+ concentration is 40 nmol/L.
2. Extracellular fluid volume: Contraction of It is also important to check the validity of results

20 Electrolyte and Acid-Base Disturbances l APICON 2009 l Jan. 29 - Feb. 1, 2009


obtained by the blood gas analyzer. For this above- to indicate purely metabolic changes. Standard
mentioned equation is used to calculate the hydrogen bicarbonate is the bicarbonate concentration in
ion concentration and the same is also derived from pH plasma in a completely oxygenated blood sample
(Table 1). If there is discrepancy between the two results, equilibrated with a pCO2 of 40 mmHg at 37C. A
the blood should be analyzed again. standard bicarbonate value below 24 mmol/L
indicates metabolic acidosis and above 24 mmol/L
Table 1: Relation between pH and Hydrogen ion
metabolic alkalosis. This conclusion is based on the
concentration
assumption that titration of whole-blood with CO2
pH 7.7 7.6 7.5 7.4 7.3 7.2 7.1 7.0 6.9 6.8 produces the same bicarbonate variations in-vitro
H+ 20 25 32 40 50 64 80 101 128 160 and in-vivo. Unfortunately, this is not always the
(nmol/L) case. Buffer base represents the total equivalent
concentration of all anionic (basic) buffer
Example: components of the blood, namely hemoglobin,
pH = 7.2, pCO2 = 30, HCO3 = 22 bicarbonate, plasma proteins and phosphates. It is
normally 48 mmol/L.
64 = 24 x 30/22
64 32 (LAB ERROR) 2. Base excess or base deficit: Base excess indicates
the deviation of base buffer from its normal value.
It can also be defined as the number of mmol of
Common Terms used in Evaluation of Acid- strong acid that is needed to adjust the pH to 7.4
Base Status when blood is equilibrated at a pCO2 of 40 mmHg.
Several terms are used in the evaluation of acid-base It is calculated from pH, paCO2 and haemoglobin.
disorder. An increase in the amount of buffer base is termed
as base excess while a decrease may be referred to
Acidaemia: A state when blood pH is below 7.36 as a base deficit or negative base excess. The BE of
Alkalaemia: A state when blood pH is above 7.44 oxygenated blood with an Hb of 15 g/dl at a pH of
Acidosis: Any disorder which adds acid or removes base 7.4 and pCO2 of 40mmHg is zero. The BE is strongly
from the body influenced by the Hb concentration, which is the
Alkalosis: Any disorder which adds alkali or removes main buffer in blood.
acid from the body. 3. Standard base excess: It was noted that changes in
(Note: In general, acidosis induces acidaemia and alkalosis pCO2 on HCO3 in vivo differed from that observed
induces alkalaemia. However, the difference between these in-vitro and that the discrepancy could be reduced
phenomenon becomes important in those patients who have by using a Hb value of 5 g/dL. This empiric value
mixed acid-base disturbances in which both acidotic and was presumed to reflect the average concentration
alkalotic processes may coexist. In this setting, the net pH of Hb of the fluid space through which bicarbonate
may be acadaemic, even though a disorder which induces distributes (between Hb of 15 g/dL in blood to Hb
an alkalosis is also present). of 0 g/dl in extracellular fluid). This was called the
Respiratory disorders: Disorders where there is standard base excess (SBE). As the SBE was
alteration in carbon dioxide concentration initially. independent of pCO2, it was used to define the
metabolic component of an acid-base disturbance.
Metabolic disorders: Disorders which affect the However, this has not been found useful in many
bicarbonate concentration initially. cases and its use is infrequent in analysis of acid-
Besides these basic terms, a standard blood gas machine base disorders.
measures certain parameters which may be required in 4. Calculated oxygen saturation (%sO 2 c): It is
some patients. These are discussed below. calculated assuming that the oxyhaemoglobin
1. Buffer base and standard bicarbonate: To recognize dissociation curve is not shifted. It will be affected
and quantify metabolic disorders, changes in plasma if carboxyhaemoglobin is also present in the sample.
bicarbonate are usually evaluated. However, since
plasma bicarbonate concentration is also affected
Anion Gap
by changes in pCO2 i.e., the respiratory disturbances,
buffer base and standard bicarbonate have been used An important tool employed in evaluating metabolic

Electrolyte and Acid-Base Disturbances l APICON 2009 l Jan. 29 - Feb. 1, 2009 21


acidosis is anion gap. It is calculated as: (Na+) - (HCO3 is a shift of H+ from the ECF to intracellular fluid while
+ Cl) and is equal to 8-12 mmol/L. This is because of potassium comes out from the cells into ECF resulting
presence of unmeasured anions. Table 2 lists commonly in hyperkalaemia. The reduced pH stimulates the central
present anions and cations which are not measured respiratory centre leading to hyperventilation and
routinely. Since major anion is albumin, AG must always reduction in carbon dioxide. This in turn reduces the
be corrected if patient has significant changes in pH produced by the initial pathology. For
hypoalbuminaemia. For every 1 g/dL albumin below 4 each 1 mEq/L decrease in bicarbonate, CO2 reduces by
g/dL, add 2.5 to the calculated serum AG. 1.2 mmHg. Winters formula for determining
compensation in metabolic acidosis is:
Table 2: Unmeasured anions and cations
Unmeasured Cations: Unmeasured Anions: pCO2 = 1.5 [HCO3] + 8 2

l Total 11 mEq/L l Total 23 mEq/L However, full compensation of pH does not occur and
in acute metabolic acidosis, the minimum CO2 which
Potassium 4 Sulphates 1
can be achieved is 10 mmHg while in chronic acidosis,
Calcium 5 Phosphates 2 it is 15 mmHg.
Magnesium 2 Albumin 16
Lactic acid 1 Pathophysiology
Org. acids 3
Metabolic acidosis can be caused by three mechanisms:
Example: 1. Increased production of acids.
A patient has following ABG values: pH 7.20; pCO2 26; 2. Decreased excretion of acids by the kidneys
HCO3 10; Na+ 139; Cl- 110. His albumin is 2 g/dL. 3. Loss of alkali from the body
The anion gap = 139 (110 + 10) = 19 mmol/L Anion gap
Albumin correction: 2.5 x 2 = 5 mmol/L
An important tool employed in evaluating metabolic
Corrected Anion Gap: 19 + 5 = 24 mmol/L acidosis is anion gap. It is calculated as: (Na+) - (HCO3
Alterations in the concentration of unmeasured ions can + Cl) and is equal to 8-12 mmol/L. This is because of
lead to misestimation of the baseline AG. As an example, presence of unmeasured anions. Increased accumulation
both hypoalbuminaemia (reduced unmeasured anions) of these unmeasured anions leads to acidosis with raised
and hyperkalaemia (increased unmeasured cations) can anion gap. In some patients of acidosis, there is exchange
lower the AG. Thus a patient with one or both of these of bicarbonate with chloride leading to loss of
disorders may have a baseline AG of 2 rather than 8 bicarbonate and hyperchlorinaemia. This leads to a
meq/L. In this setting, an AG of 14 meq/L, which is normal anion gap with acidosis.
only mildly above the normal limit, represents a true It is important to calculate AG in all patients when acid-
elevation in the AG of as much as 12 meq/L. base interpretation is being done. An AG > 12 mmol/L
can indicate metabolic acidosis while an AG > 20 mmol/
Metabolic Acidosis L always indicates metabolic acidosis.
Metabolic acidosis is initiated by a reduction in plasma Diabetic ketoacidosis is typically associated with an
bicarbonate concentration. A pH < 7.2 may be life elevated AG. If renal function and volume status is
threatening because (i) the enzyme systems become well maintained, however, some or many of the excess
unreliable, (ii) electrolyte concentration is altered, (iii) ketone anions may be excreted in the urine as the
electrophysiological functions of the body become sodium and potassium salts. The net effect is that the
unreliable and this includes impaired cardiac rise in the AG may be much less than expected from
contractility and reduced threshold for ventricular the severity of metabolic acidosis. Furthermore, the loss
fibrillation, and (iv) the autonomic responses to drugs of these organic anions is equivalent to the loss of
are altered. bicarbonate, since metabolism of ketoacid anions
results in regeneration of bicarbonate. Thus a normal
Compensation in metabolic acidosis AG acidosis is typically seen during the treatment phase
of DKA due to the urinary loss of these bicarbonate
Because of increased H+ ion concentration in ECF, there precursors.

22 Electrolyte and Acid-Base Disturbances l APICON 2009 l Jan. 29 - Feb. 1, 2009


Aetiology volume of breath. This can produce respiratory fatigue
in seriously ill patients. Other non-specific features
Important causes of metabolic acidosis are listed in
include fatigue, confusion, coma, and cardiovascular
Table 3.
features due to vasodilatation and reduced cardiac
Table 3: Causes of metabolic acidosis contractility which may lead to shock and cardiac failure.
Increased Anion Gap Presence of hyperkalaemia (due to shift of intracellular
potassium) may mask significant loss of potassium from
1. Severe renal failure
body stores. Presence of hypokalaemia in metabolic
2. Increased production of organic acids acidosis indicates severe potassium loss from the body.
a. Diabetic ketoacidosis
b. Alcoholic ketosis Diagnosis
c. Starvation ketosis The first step is to calculate whether the patient has
d. Poisonings: Methanol, salicylates, ethylene adequate respiratory compensation or whether he has
glycol, carbon monoxide, cyanide associated other acid-base disorders. The next step is to
e. Increased lactic acid production: calculate anion gap, delta-delta ratio and osmolar gap.
Urinary anion gap (UAG) may be of some use in
Cardiorespiratory arrest
differential diagnosis.
Convulsions
Shock Delta-Delta ratio or delta ratio
Septicaemia Delta ratio = (Increase in Anion Gap / Decrease in
Liver failure bicarbonate)
Normal anion gap (Hyperchlorinaemia)
The normal value is between 1 and 1.6. A low delta gap
1. Diarrhea (Loss of alkali) suggests the presence of a concomitant non-gap acidosis,
2. Potassium sparing diuretics (renal tubular whereas a delta gap greater than 1.6 suggests the
dysfunction) presence of a concomitant metabolic alkalosis.
Another way to remember various causes is given below It might be assumed that there is a 1:1 relationship
(Table 4) between the increase in AG (rAG) and the decrease in
plasma bicarbonate (rHCO3). However, the r/r ratio
Table 4. Causes of Metabolic Acidosis is typically greater than 1 in lactic acidosis, since the
High Anion Metabolic Non-anion Gap Metabolic space of distribution of the hydrogen ion and the lactate
Acidosis (MUDPILES-R) Acidosis (HARDUPS) anion are different. Most of the lactate anions remain in
the extracellular fluid, thereby raising the AG. In
l Methanol l Hyperalimentation
comparison, more than 50 per cent of the excess
l Uraemia l Acetazolamide, hydrogen ions is buffered in the cells and in bone in
amphotericin mild metabolic acidosis, a process that does not lower
l Diabetic ketoacidosis l Renal tubular acidosis the plasma bicarbonate concentration.Thus, the rise in
l Paraldehyde l Diarrhoea anion gap tends to exceed the reduction in the plasma
l Iron, INH l Ureteral diversions bicarbonate concentration in lactic acidosis.
l Lactic acidosis l Pancreatic fistula In ketoacidosis, the r/r ratio averages about 1:1. In
l Ethylene glycol l Saline resuscitation this disorder, the loss of ketoacid anions in the urine
l Salicylates lowers the AG without affecting the plasma bicarbonate
l Rhabdomyolysis concentration, therefore tending to balance the effect
of intracellular buffering of hydrogen. The amount of
Clinical features ketoacid anions excreted in ketoacidosis depends on the
degree to which glomerular filtration is maintained.
Features of metabolic acidosis are non-specific. The Patients with impaired renal function (due to underlying
patients usually hyperventilate due to stimulation of diabetic nephropathy or volume depletion) will retain
respiratory centre. This is called Kussmauls breathing the ketoacid anions and have a relatively high anion
that is characterised by increased rate as well as tidal gap in relation to the fall in the plasma bicarbonate

Electrolyte and Acid-Base Disturbances l APICON 2009 l Jan. 29 - Feb. 1, 2009 23


concentration, similar to that in lactic acidosis. (exception is type 2 RTA). UAG is not useful in oliguria
and hypovolaemia.
Delta gap
Urine pH
Another term used in the analysis of acid-base disorder
is the delta gap. In pure high anion gap metabolic In non-anion metabolic acidosis, urine pH may be useful
acidosis, for every 1 mEq/L rise in anion gap, there if the source of loss of fluids is not from intestines. If
should be a concomitatnt fall of 1 meq/l in bicarbonate. urine pH > 5.5, type 1 renal tubular acidosis is likely. If
Any significant deviation from this relationship suggests urine pH < 5.5, check serum potassium. If serum
presence of a mixed acid-base disorder. This can be potassium is low, RTA type 2 is likely; if serum potassium
expressed as the delta gap. It equals deviation in anion is high, type 4 RTA is likely.
gap from normal minus deviation in bicarbonate from
normal. The usual range of delta gap is -6 to +6 and in Treatment
case of a pure high anion gap metabolic acidosis, it
should be zero. If the calculated delta gap is > 6, this Whenever possible, efforts should be directed at
suggests that the bicarbonate has not dropped to the identifying and treating the underlying process which
expected extent and thus there is concomitant netabolic gave rise to metabolic acidosis. When metabolic acidosis
alkalosis or rarely respiratory acidosis. A delta gap of results from inorganic acids (i.e., hyperchloraemic or
lesser than 6 implies a greater loss of bicarbonate than normal anion gap acidosis), HCO3 is required to treat
can be accounted for by the metabolic acidosis and the acid-base disturbance. However, when acidosis
suggests a concurrent normal anion gap metabolic results from organic acid accumulation (ie, increased
acidosis or rarely respiratory alkalosis. However, the anion gap acidosis), as in lactic acidosis, ketoacidosis,
information gained from delta gap is same as that gained or the intoxication syndromes, the role of NaHCO3 is
from delta-delta ratio. controversial. Use of intravenous sodium bicarbonate
should be restricted to patients with severe acidosis i.e.,
Osmolar gap bicarbonate < 6 mEq/L.

Osmolality of a solution is the number of osmoles of In normal individuals, approximately equal amounts of
solute per kilogram of solvent. Osmolarity of a solution H+ are buffered by ECF bicarbonate and ICF buffers.
is the number of osmoles of solute per litre of solution. Therefore, half of the administered bicarbonate will
The osmolar gap is the difference between the measured accept H+ from ICF and will be destroyed. Hence, the
osmolality and the calculated osmolarity. dose of bicarbonate required = Desired increase in
plasma bicarbonate x 40% of body weight (40% is double
Calculated osmolarity: 2 (Na+) + glucose/18 + BUN/ the ECF volume). This volume of 40% of body weight is
2.8 called bicarbonate space. In severe acidosis, bicarbonate
Normal osmolar gap is < 10 mOsm/L. It is elevated in space may be large due to intracellular shift of H+ and
several conditions including poisoning with ethanol, buffering of H+ by bone and cellular buffers. Therefore,
methanol, and ethylene glycol. frequent determinations of acid-base status are required.
Half of the calculated bicarbonate should be
Urinary anion gap administered first and acid-base estimation is repeated
after that before further bicarbonate is administered.
In normal circumstances, urine is free of HCO3. Spot
urine electrolytes are measured to calculate UAG. NH4 Example:
is the predominant unmeasured cation in urine. It is
accompanied by Cl-. A patient weighing 70 kg has following blood gas values:
pH = 7.1; pCO2 = 20 mmHg; HCO3 = 6 meq/L
UAG = Na+ + K+ - Cl-
Normal UAG = - 20 to positive The pH should be raised to 7.2. At the pH of 7.2, the H+
concentration will be 63 nmol/L. Therefore, using the
If non-renal source of non-anion gap metabolic acidosis modified Henderson-Hasselbach formula,
is present (e.g., diarrhea), there is dramatic increase in
NH4 excretion in urine and therefore UAG becomes more 63 = 24 x 20/HCO3 or HCO3 = 8 meq/L.
negative ( < 20 and may even become -50). If patient Presuming bicarbonate space of 70%, the amount of
has renal tubular acidosis (RTA), UAG is positive bicarbonate required to increase pH to 7.2 will be:

24 Electrolyte and Acid-Base Disturbances l APICON 2009 l Jan. 29 - Feb. 1, 2009


= Desired increase in bicarbonate x bicarbonate space THAM: Another carbon dioxide-consuming alkalinizing
x weight agent is THAM, a commercially available solution of
= 2 x 70/100 x 70 = 98 mEq 0.3 N-tromethamine. This sodium-free solution buffers
both metabolic acids (THAM + H+ = THAM+) and
Initially, half of this i.e., 50 mEq should be administered respiratory acids (THAM + H 2CO 3 = THAM + +
intravenously. If situation is not very urgent, it may be HCO3). Like Carbicarb, THAM limits carbon dioxide
administered over a period of 15 minutes. generation and increases both extracellular and
Dangers of bicarbonate: There are some dangers of intracellular pH. Nevertheless, THAM has not been
bicarbonate therapy, particularly when administered in documented to be clinically more efficacious than
excess, and due to these limitations, bicarbonate therapy bicarbonate. In fact, serious side effects, including
is at present not recommended in the initial phase of hyperkalemia, hypoglycemia, ventilatory depression,
resuscitating patients with cardiorespiratory arrest. local injury in cases of extravasation, and hepatic
Various dangers of bicarbonate therapy are: necrosis in neonates, markedly limit its usefulness.
1. Hypokalaemia (due to shift of potassium
intracellularly) Metabolic Alkalosis
2. Tetany (due to alkalosis) Metabolic alkalosis is usually initiated by an increased
loss of acid from stomach or kidneys. Hypokalaemia
3. Congestive heart failure due to sodium overload occurs because of shift of potassium into intracellular
4. Respiratory alkalosis: Due to reaction of area.
administered bicarbonate with H+ ions, carbon
dioxide is liberated which diffuses into the brain and Compensation
produces acidosis of cerebrospinal fluid. This Alkalosis gives rise to hypoventilation and hence
stimulates the respiratory centre leading to retention of CO2 which attenuates the increase in
respiratory alkalosis. systemic pH. The compensatory response is that for each
5. Metabolic alkalosis can occur due to excessive 1 mEq/L increase in bicarbonate, CO2 increases by about
administration of bicarbonate. Another cause of 0.6 mmHg. Even in the absence of underlying lung
metabolic alkalosis is that with the successful disease, secondary hypoventilation is accompanied by
treatment of primary disorder, there is rapid some degree of hypoxia.
conversion of lactate and ketones into bicarbonate
which may lead to alkalosis. Pathophysiology

6. Intracellular acidosis may occur because of Under normal conditions, kidneys excrete excessive
intracellular diffusion of liberated carbon dioxide. bicarbonate rapidly so that alkalosis will not be sustained
Intracellular acidosis may hamper the functions of unless bicarbonate reabsorption in the kidneys is
metabolic enzymes. enhanced or alkali is generated at a great rate. The
maintenance of alkalosis is due to three factors:
Other options 1. Stimulation of bicarbonate reabsorption due to
Carbicarb: It consists of equimolar concentrations of volume depletion when renal conservation of
sodium bicarbonate and sodium carbonate. Because sodium takes precedence over correction of
carbonate is a stronger base, it is used in preference to alkalosis. Since a large fraction of plasma sodium is
bicarbonate for buffering hydrogen ions, generating bound with bicarbonate, complete absorption of
bicarbonate rather than carbon dioxide in the process filtered sodium requires reabsorption of bicarbonate
(CO3 + H+ = HCO3). In addition, the carbonate ion as well. Alkalosis is sustained till volume depletion
can react with carbonic acid, thereby consuming carbon exists. On administering sodium chloride, tubular
dioxide (CO3 + H2CO3 = 2HCO3). Thus, Carbicarb avidity for sodium decreases. Also, chloride is
limits but does not eliminate the generation of carbon available as an alternative to bicarbonate.
dioxide. However, the risks of hypervolemia and 2. Another mechanism which can maintain alkalosis
hypertonicity are similar to bicarbonate, and neither is hypermineralocorticoidism which stimulates
agent prevents progressive reduction in myocardial-cell increased renal H+ secretion as ammonium and
pH in animals with ventricular fibrillation. titrable acidity. The patients are not volume-depleted

Electrolyte and Acid-Base Disturbances l APICON 2009 l Jan. 29 - Feb. 1, 2009 25


or chloride-dependent and therefore, do not respond on ECG, and increased susceptibility to dysrrhythmias.
to saline infusion. These are either due to hypokalaemia or alkalosis per
se. Hypoventilation due to compensation may be
3. The third mechanism of sustaining alkalosis is severe
significant in patients with compromised respiration.
potassium depletion as renal conservation of
potassium takes precedence over H+ resulting in loss
of H+ in the urine and hence acidic urine in presence Treatment
of alkalosis. Mild-to-moderate alkalosis rarely requires any specific
Based on above, various causes of metabolic alkalosis treatment. In presence of severe, saline-responsive
are listed in Table 5. alkalosis, infusion of saline and potassium is sufficient
to enhance renal excretion of bicarbonate. The ensuing
Table 5: Causes of metabolic alkalosis increase in NaHCO3 excretion is due primarily to two
Saline Responsive (Urinary Na+ and Cl < 10 meq/L) events occurring in the collecting tubules decreased
reabsorption and increased secretion of bicarbonate.
1. Loss of chloride-rich, bicarbonate-poor sweat (cystic
Repletion of the extracellular volume removes the
fibrosis)
stimulus to sodium retention, thereby allowing less
2. Diuretics (volume contraction)
bicarbonate to be reabsorbed. The increase in distal
3. Use of poorly reabsorbable anions like carbenicillin chloride delivery will raise the chloride concentration
(loss of H+). in the tubular lumen. This will create a more favorable
4. Post-hypercapneic alkalosis gradient for chloride entry into the cells, thereby
5. Gastric alkalosis allowing increased bicarbonate secretion in the cortical
Saline Un-responsive (Urinary Na+ and Cl > 15-20 collecting tubule.
meq/L) In saline-resistant cases, specific treatment of primary
1. Severe potassium depletion due to any cause disorder is required. Hypokalaemia should be corrected.
2. Primary aldosteronism and Cushings syndrome Spironolactone inhibits secretion of H+ and hence may
correct alkalosis. In patients not responding to these
In patients with chronic hypercapnoea due to respiratory
therapeutic measures, acetazolamide may be used.
insufficiency, the plasma bicarbonate is high due to renal
compensation. If the primary condition improves (as In severe cases, intravenous acidifying agents (dilute
with ventilatory support), pCO2 falls promptly. However, solutions of arginine hydrochloride, lysine
urinary excretion of excessive bicarbonate takes a hydrochloride, and hydrogen chloride) may be required.
number of days. In patients who are on low salt diet or Hydrochloric acid administered intravenously as a 0.1
on diuretic and are therefore volume depleted, post- to 0.2 N solution (that is, one containing 100 to 200
hypercapneic alkalosis will persist as kidneys will not mmol of hydrogen per liter) is safe and effective for the
be able to excrete bicarbonate from the body. management of severe metabolic alkalosis. The acid can
be infused as such or can be added to amino acid and
Patients who are having repeated vomiting or are on
dextrose solutions containing electrolytes and vitamins
continuous gastric aspiration, there is loss of acid leading
without causing adverse chemical reactions. Because of
to initiation of alkalosis. However, volume and
its sclerosing properties, hydrochloric acid must be
potassium depletion both tend to maintain alkalosis.
administered through a central venous line at an infusion
rate of no more than 0.2 mmol per kilogram of body
Clinical features weight per hour. However, it can also be administered
There are no specific manifestations of metabolic through a peripheral vein if it is added to an amino acid
alkalosis. Arteriolar constriction can produce reduced solution and mixed with a fat emulsion. Calculation of
cerebral and coronary blood flow. Severe alkalosis may the amount of hydrochloric acid solution to be infused
cause apathy, confusion or stupor. The patients may have is based on a bicarbonate space of 50 percent of body
increased neuromuscular irritability due to increased weight. Thus, to reduce plasma bicarbonate from 50 to
binding of calcium to proteins thereby reducing ionized 40 mmol per liter in a 70-kg patient, the estimated
calcium and also because of increased acetylcholine amount of hydrochloric acid required is 10 x 70 x 0.5,
release at the neuromuscular junctions. This may or 350 mmol. Precursors of hydrochloric acid, such as
produce tetany and seizures. The cardiac manifestations ammonium chloride (20 g per liter, with 374 mmol of
are increased QT interval and prominence of U waves hydrogen per liter) and arginine monohydrochloride

26 Electrolyte and Acid-Base Disturbances l APICON 2009 l Jan. 29 - Feb. 1, 2009


(100 g per liter, with 475 mmol of hydrogen per liter), 2. Chronic
can substitute for hydrochloric acid, but they entail a. Chronic obstructive lung disease
substantial risks and are used less commonly. Both of
Disorders of Musculo-skeletal System
these preparations are hyperosmotic solutions; to avoid
local tissue injury, they must be infused through a central 1. Acute
catheter. In addition, ammonium chloride can raise a. Flail chest
serum ammonia concentrations in patients with liver b. Tension pneumothorax
failure, and arginine monohydrochloride can induce c. Use of aminoglycosides
serious hyperkalemia in patients with renal failure, 2. Chronic
especially when there is coexisting liver disease. a. Myasthenia gravis
b. Poliomyelitis
Respiratory Acidosis Respiratory Centre Abnormalities
Hypoventilation due to any cause leads to prompt 1. Acute
increase in pCO2 leading to respiratory acidosis. Because a. Overdose with opiates and sedatives
lungs are so efficient at expiring CO2, any abnormal b. Use of general anesthetic agents
increase in carbon dioxide can directly be attributed to c. Cardiac arrest
an abnormality affecting this organ system.
2. Chronic
Compensation a. Central nervous system lesions

In the ECF, there is no buffer to bind CO2 as bicarbonate Clinical features


cannot bind it. In acute respiratory acidosis, the only
buffers available are intracellular proteins and It is often difficult to separate manifestations of
bicarbonate formed as a result of intracellular buffering respiratory acidosis from those of associated hypoxia
diffuses out of cells into ECF increasing its bicarbonate and hypercapnia. Because of hypercapnia, patients may
levels and reducing change in pH. In chronic acidosis, have asterixis and they may be obtunded or confused.
renal acid secretion is enhanced and bicarbonate is They have increased sweating. Fundus examination may
reabsorbed. The response occurs because increased show papilledema.
arterial pCO2 increases intracellular pCO2 in proximal
tubular cells and this causes increased H+ secretion from Treatment
the tubular cells into the tubular lumen. Chloride is The only treatment for severe respiratory acidosis is
secreted in place of bicarbonate leading to correction of underlying disorder and use of assisted
hypochlorinemic acidosis. Due to these changes, in acute ventilation. Rapid infusion of alkali in the early phase
and chronic respiratory acidosis, the bicarbonate of cardiac arrest is not recommended due to various
increases by 1 mEq/L and 3.5 mEq/L respectively. complications of bicarbonate therapy.
Pathophysiology
Respiratory Alkalosis
Any process which decreases ventilation leads to
Respiratory acidosis is due to hyperventilation leading
respiratory acidosis. Important causes of respiratory
to decrease in carbon dioxide. Because carbon dioxide
acidosis are listed in Table 6.
is excreted by the lungs, the only cause of respiratory
Table 6: Causes of respiratory acidosis alkalosis is hyperventilation.
Disorders of Gas Exchange
Compensation
1. Acute
a. Severe asthma In acute respiratory alkalosis, reduction of CO2 leads to
b. Acute exacerbation of obstructive lung disease release of H+ from intracellular buffers which decrease
plasma bicarbonate levels and hence reduce pH changes.
c. Late stages of pulmonary edema
In chronic hypocapnia, plasma bicarbonate is further
d. Laryngospasm reduced as decreased CO2 inhibits renal reabsorption of
e. Foreign body inhalation bicarbonate. In acute and chronic respiratory alkalosis,
f. Malfunctioning ventilators the bicarbonate decreases by 2 mEq/L and 5 mEq/L for

Electrolyte and Acid-Base Disturbances l APICON 2009 l Jan. 29 - Feb. 1, 2009 27


every 10 mmHg decrease in CO2. Clinical features
The clinical manifestations of respiratory alkalosis
Pathophysiology
depend on the severity and acuteness of alkalosis. In
Hyperventilation can be due to systemic disorders or acute cases, hyperventilation is evident and the patient
primary lung disorders (Table 7). complains of paresthesias, numbness, tingling and in
some cases, tetany. In severe cases, confusion and coma
Table 7: Causes of respiratory alkalosis
can occur due to cerebral vasospasm produced by
Central Mechanisms hypocapnea.
1. Anxiety
2. Fever Treatment
3. CNS infections
The mainstay of treatment is control of underlying
4. Cerebrovascular accident disease. In acute hyperventilation syndrome due to
5. Metabolic encephalopathy anxiety, with syncope and tetany, sedation and
6. Septicemia rebreathing are required.
7. Salicylate poisoning
8. Cirrhosis of liver Primary and Compensatory Changes in Acid-
Pulmonary Mechanisms Base Disorders
1. Interstitial lung disease
Table 8 and 9 summarize the primary and
2. Asthma
compensatory changes in various disorders of acid-base
3. Pneumonia imbalance.
4. Congestive heart failure
5. Pulmonary embolism

Table 8: Primary and compensatory responses in various acid-base disorders


Disorder pH Primary response Compensatory response
Metabolic acidosis Decreased Decreased HCO3 Decreased pCO2
Metabolic alkalosis Increased Increased HCO3 Increased pCO2
Respiratory acidosis Decreased Increased CO2 Increased HCO3
Respiratory alkalosis Increased Decreased CO2 Decreased HCO3

Table 9: Compensatory responses in acid-base disorders


Disorder Compensation Limit of compensation
Metabolic acidosis For every 1 mEq/L fall in HCO3, pCO2 does not fall below
pCO2 falls by 1.2 mmHg 10 mmHg
Metabolic alkalosis For every 1 mEq/L rise in HCO3, pCO2 does not rise above
pCO2 rises by 0.6 mmHg 55 mmHg
Acute respiratory For every 10 mmHg rise in pCO2, HCO3 does not rise above
acidosis HCO3 rises by 1 mEq/L 30 mEq/L
Chronic respiratory For every 10 mmHg rise in pCO2, HCO3 does not rise above
acidosis HCO3 rises by 3.5 mEq/L 45 meq/L
Acute respiratory For every 10 mmHg fall in pCO2, HCO3 does not fall below
alkalosis HCO3 falls by 2 mEq/L 18 mEq/L
Chronic respiratory For every 10 mmHg fall in pCO2, HCO3 does not fall below
alkalosis HCO3 falls by 5 mEq/L 12 mEq/L

28 Electrolyte and Acid-Base Disturbances l APICON 2009 l Jan. 29 - Feb. 1, 2009


Mixed Acid-Base Disorders respiratory alkalosis. Presence of sepsis may point
towards mixed metabolic acidosis and respiratory
Disorders of mixed acid-base imbalance are quite alkalosis. Further, acid-base abnormalities should be
common in sick patients as these patients may be on suspected in patients who are critically ill, are
multiple drugs and ventilatory support. Important experiencing extreme dyspnoea, or have abnormal
causes of mixed acid-base disturbances are shown in mental status.
Table 10.
Table 10: Causes of mixed acid-base disorders Step 2: Acidemia or alkalemia. Whether values suggest
academia (pH < 7.38) or alkalemia (pH > 7.42).
Respiratory Acidosis with Metabolic Acidosis
1. Cardiac arrest Step 3: Identify primary disorder. Look at pH, pCO2
2. Severe pulmonary edema and HCO3. If the pH is below 7.38, and both pCO2 and
3. Drug poisoning with severe respiratory depression bicarbonate are low, metabolic acidosis is present. If pH
is below 7.38 and both pCO2 and bicarbonate are high,
Respiratory Alkalosis with Metabolic Alkalosis it indicates presence of respiratory acidosis.
1. Hepatic failure with diuretic use
If the pH is higher than 7.42, alkalosis is present. Again
2. Patients on ventilator and nasogastric suction measure pCO2 and bicarbonate. If both carbon dioxide
Mixed Acute and Chronic Respiratory Acidosis and bicarbonate are high, metabolic alkalosis is present.
1. Chronic obstructive airway disease with If both carbon dioxide and bicarbonate are low,
superimposed infection respiratory alkalosis is present.

Respiratory Alkalosis with Metabolic Acidosis Step 4: Calculate compensatory response. The formulas
1. Septic shock are listed in table 9. In acute metabolic acidosis for
2. Salicylate overdose example, Winters Formula can be used to calculate
whether the respiratory compensation is appropriate.
3. Renal failure with sepsis In metabolic acidosis, pCO2 should be equal to 1.5 times
Respiratory Acidosis with Metabolic Alkalosis bicarbonate levels plus 8 2. If the patients pCO2 is
1. COAD with diuretic use higher, the patient has a superimposed respiratory
acidosis. If the pCO2 is below this range, the patient has
Metabolic Acidosis with Metabolic Alkalosis a superimposed respiratory alkalosis.
1. Diarrhea and vomiting
2. Metabolic acidosis with excessive bicarbonate Step 5: Calculate anion gap. Presence of high anion gap
therapy indicates high anion-gap metabolic acidosis.

Step 6. Calculate delta-delta ratio.


Summary
DeltaDelta = Measured AG Normal AG
Fig. 4 Shows approach to interpretation of acid-base
disorders. Normal [HCO3] Measured [HCO3]
Delta-delta ratio < 1 indicates associated non-anion gap
Step-by-Step Approach to Interpret metabolic acidosis. A delta-delta ration > 1.6 indicates
associated metabolic alkalosis.
Step 1: Comprehensive history and examination.
Comprehensive history taking and physical examination
Step 7. Calculate osmolar gap. (if metabolic acidosis is
can often give clues as to the underlying acid-base
detected). A high osmolar gap indicates presence of one
disorder. For example, patients with hypotension, severe
of the alcohols.
diarrhea or renal failure are likely to have metabolic
acidosis. Patients with recurrent vomiting, nasogastric
Step 8. Formulate differential diagnosis. What is the
aspiration or on diuretics may have metabolic alkalosis.
cause of acid-base disorder?
Similarly, patients with chronic obstructive lung disease
and flail chest are likely to have respiratory acidosis
while those with pulmonary embolism may have

Electrolyte and Acid-Base Disturbances l APICON 2009 l Jan. 29 - Feb. 1, 2009 29


Fig. 4: Interpretation of Arterial Blood Gas Report.

Lastly, one case scnario will be discussed to give step-by-step approach to acid-base interpretation.

Practice Case
l 60 year male presents to the ED with rapid breathing l Step 1: Is there an acidemia or alkalemia?
and less responsive than usual. No other history is Acidemia
available
ABG: pH 7.31 Chem: Na+ 123
ABG: pH 7.31 Chem: Na+ 123
PCO2 10 K+ 5.0
+
PCO2 10 K 5.0
HCO3 5 Cl 99

HCO3 5 Cl 99
HCO3 5
HCO3 5

30 Electrolyte and Acid-Base Disturbances l APICON 2009 l Jan. 29 - Feb. 1, 2009


l Step 2. Is the primary process metabolic or l Step 6: If high anion gap acidosis, calculate delta-
respiratory? delta ratio
PCO2 = 10 should drive pH
DeltaDelta = Measured AG Normal AG
HCO3 = 5 should drive pH
Normal [HCO3] Measured [HCO3]
ABG: pH 7.31 Chem: Na+ 123
Delta-Delta = (19 10)/(24 5) = 0.47
PCO2 10 K+ 5.0
A Non-anion Gap Metabolic Acidosis also
HCO3 5 Cl 99
HCO3 5 ABG: pH 7.31 Chem: Na+ 123
PCO2 10 K+ 5.0
HCO3 5 Cl 99
l Step 3: Choose appropriate compensation formula: HCO3 5

In Metabolic acidosis, pCO2 = 1.5 [HCO3] + 8 2


ABG: pH 7.31 Chem: Na+ 123 l Step 7: If high anion gap acidosis, calculate Osmolar
PCO2 10 K+ 5.0 gap
HCO3 5 Cl 99
Osmolar gap = Measured Osmolality
HCO3 5 Calculated Osmolarity
Calculated Osmolarity = 2 (Na) + (Glucose/18) +
(BUN/2.8)
l Step 4: Is the respiratory compensation adequate?
ABG: pH 7.31 Chem: Na+ 123
Metabolic acidosis pCO2 = 1.5 [HCO3] + 8 2 PCO2 10 K+ 5.0
Expected pCO2 = 1.5 (5) +8 2 = [13.5 17.5] HCO3 5 Cl 99
PCO2 = 10, additional respiratory alkalosis HCO3 5

ABG: pH 7.31 Chem: Na+ 123


PCO2 10 K+ 5.0
HCO3 5 Cl 99
HCO3 5

l Step 5: Calculate the anion gap:

Anion gap = [Na+] [(Cl) + (HCO3]


Anion gap = 123 (99 + 5) = 19
Anion Gap Metabolic Acidosis
ABG: pH 7.31 Chem: Na+ 123
PCO2 10 K+ 5.0
HCO3 5 Cl 99
HCO3 5

Electrolyte and Acid-Base Disturbances l APICON 2009 l Jan. 29 - Feb. 1, 2009 31


Faculty Council of Indian College of Physicians

Chairman Dean
SK Bichile, Mumbai (2009) AK Das, Pondicherry (2010)

Dean Elect Vice Deans Past Dean


BBThakur, Muzaffarpur (2010) SK Goyal, Kota (2010) YP Munjal, New Delhi (2010)
OP Kalra, New Delhi (2010)
Anil Chaturvedi, New Delhi (2010)

Hon. General Secretary


Sandhya Kamath, Mumbai (2010)

Jt.Secretary (HQ) Jt.Secretary (Deans place) Hon.Treasurer


Falguni S Parikh, Mumbai (2010) TK Dutta, Pondicherry (2010) MilindY Nadkar, Mumbai (2009)

Elected Members
S Chugh, Delhi (2009) Subhash Chandra, New Delhi (2009)
MP Shrivastava, New Delhi (2009) BN Jha, Muzaffarpur (2009)
HM Lal, Hyderabad (2009) S Chandrasekharan, Chennai (2010)
Vinay Goel, New Delhi (2010) RK Goyal, Ajmer (2010)
Rajat Kumar, New Delhi (2010) MP Singh, Ranchi (2010)

Ex-Officio Members
President Elect Hon.Editor, JAPI
AK Agarwal Shashank R.Joshi
New Delhi (2009) Mumbai (2009)

Vous aimerez peut-être aussi